LSAT and Law School Admissions Forum

Get expert LSAT preparation and law school admissions advice from PowerScore Test Preparation.

 Administrator
PowerScore Staff
  • PowerScore Staff
  • Posts: 8919
  • Joined: Feb 02, 2011
|
#35109
Complete Question Explanation

Point at Issue—#%. The correct answer choice is (E)

Your task in this Point at Issue question is to select the answer choice that best describes what the
physician and the trampoline enthusiast disagree about. Their arguments, reordered for clarity,
proceed:

Physician

..... Premise: ..... there were approximately 83,400 trampoline-related injuries last year

..... Sub-Conclusion: ..... thus, trampolines are quite dangerous

..... Conclusion: ..... so, trampolines should be used only under professional supervision

Trampoline Enthusiast

..... Premise: ..... in the past ten years sales of home trampolines have increased much more
..... ..... ..... ..... than trampoline-related injuries have: 260 percent in sales compared with 154
..... ..... ..... ..... percent in injuries

..... Premise: ..... every exercise activity carries risks, even when carried out under professional
..... ..... ..... ..... supervision

..... Conclusion: ..... thus, it is not the case that trampolines should be used only under professional
..... ..... ..... ..... supervision

The correct answer choice in this Point at Issue question will describe the information concerning
which the physician and the trampoline enthusiast disagree with each other. The incorrect answers
will present information either not discussed by at least one of the speakers, or concerning which the
speakers would agree.

Answer choice (A): The trampoline enthusiast does not express the opinion that trampolines cause
injuries. Rather, she spoke only of trampoline-related injuries.

Answer choice (B): The physician expressed no opinion regarding home trampolines.

Answer choice (C): The physician provided only the number of trampoline-related injuries last year,
and did not discuss the rate of trampoline-related injuries.

Answer choice (D): Neither speaker expressly states that professional supervision of trampoline
use tends to reduce injuries from the activity. The enthusiast’s comments come closest to this idea,
suggesting that the risk associated with the activity may be lessened by professional supervision.
The physician, however, does not indicate that the professional supervision would reduce the risk of
injury, which the physician ascribes to the nature of the trampoline itself.

Answer choice (E): This is the correct answer choice. The physician expressly states this
conclusion, to which the enthusiast replies, “I disagree.”
 15veries
  • Posts: 113
  • Joined: Sep 25, 2016
|
#29917
Hi,

I was not sure why D is wrong and E is right.

D...I thought they disagree this because of the last sentence of Trampoline entusiast. He did not say this directly, but I thought we can assume this idea.
I thought supervision
E...I was not sure Trampoline enthusiast's idea about "mandatory" supervision. So unlike D I thought we should not assume this idea...
So I guess my question is when we can assume what implicit idea included in their explicit words...
 Claire Horan
PowerScore Staff
  • PowerScore Staff
  • Posts: 408
  • Joined: Apr 18, 2016
|
#30226
The correct answer will be the answer choice for which one person agrees with the statement and the other person disagrees. Find in the passage where a person states the same, or the opposite, opinion.

(A) trampolines cause injuries to a significant number of people using them (Physician and Enthusiast agree)
(B) home trampolines are the main source of trampoline-related injuries (Neither says this, so we don't know whether they would agree or not)
(C) the rate of trampoline-related injuries, in terms of the number of injuries per trampoline user, is declining (The Physician's comment do not address whether there is a trend in the rate of trampoline-related injuries, so we don't know.)
(D) professional supervision of trampoline use tends to reduce the number of trampoline-related injuries (The Enthusiast says that the risk would be greater than zero even with professional supervision, but doesn't say whether the risk would be reduced. We don't know if they would agree or disagree on this.)
(E) trampoline use is an activity that warrants mandatory professional supervision (This is where the Physician and the Enthusiast disagree. The Physician says that the number of trampoline-related injuries "suggests that trampolines are quite dangerous and should therefore be used only under professional supervision." The Enthusiast quibbles with the classification as "quite dangerous," asserting that injury rates have declined. The Enthusiast is thereby questioning the Physician's intermediate conclusion. The Enthusiast continues, arguing that exercise activities always have risks even with supervision. The only purpose the Enthusiast could have for saying this would be to imply that professional supervision shouldn't be required for trampolining.
User avatar
 RichieC
  • Posts: 9
  • Joined: May 11, 2021
|
#89160
Hi,

Is it really okay to assume that when the enthusiast disagrees, they are disagree with the conclusion immediately before it? I feel like this could be an assumption used for a trick question, where the enthusiast might mean they disagree with the Physician's last sentence as a whole or they disagree with the intermediate conclusion.

I'm also not sure that disagreeing with the intermediate conclusion means disagreeing with the final conclusion either. Is there a reason for doing this? I feel as though the enthusiast never even implies that trampoline use does not warrant supervision.
 Robert Carroll
PowerScore Staff
  • PowerScore Staff
  • Posts: 1787
  • Joined: Dec 06, 2013
|
#89168
Richie,

It's not OK to assume that! There's no assumption here. We're analyzing what the trampoline enthusiast said to pinpoint where the disagreement is. We know they disagreed, we just need to determine what the disagreement was about.

It can't be about the first sentence. The enthusiast doesn't say anything about the number of injuries.

So it has to be something in the second sentence. There are two claims in that sentence - it's dangerous and it should be used only under professional supervision.

The enthusiast provides information that is fairly consistent with trampolines being dangerous, so the goal of the enthusiast is not straightforwardly to try to prove they're not dangerous. Instead, the enthusiast, while acknowledging they carry some danger, thinks that danger is overstated, and that all exercises carry risks. This claim about all exercises is basically claiming that trampolines aren't so much more dangerous than other exercises that they should be singled out. In what way ARE they being singled out? They're being singled out as requiring professional supervision. The enthusiast's goal is to show that the last statement the physician made is false, so that's the point on which they're disagreeing.

Note that if, instead, you think the enthusiast's only disagreement is with "trampolines are quite dangerous", no answer choice fits. That is a decisive hint that that's, at best, only part of the disagreement. As I said, the first sentence isn't a point at issue either, so there's only one thing left, and only answer choice (E) talks about it.

Robert Carroll

To the extent that
User avatar
 RichieC
  • Posts: 9
  • Joined: May 11, 2021
|
#89200
Hi Robert,

Thank you, I think your explanation works well for me. So it seems that in this case we manage to pinpoint the disagreement by eliminating two of the three statements by the physician. I also did not notice that "trampolines are quite dangerous" is not an answer choice.

Thanks again,
 sofisofi
  • Posts: 23
  • Joined: Mar 31, 2022
|
#97363
Hi I'm having a hard time understanding why A is wrong. To me it seems like the enthusiast is citing the the increase of home trampolines relative to the increase in the number of trampoline-related injuries as a way to say trampolines don't cause a significant number of injuries. It also seems like the physician disagrees with this when he cites in the beginning that there were approximately 84,300 trampoline related injuries last year.
Any clarification is much appreciated!
 Adam Tyson
PowerScore Staff
  • PowerScore Staff
  • Posts: 5153
  • Joined: Apr 14, 2011
|
#97877
As I see it, sofisofi, the Physician does agree that a significant number of people are being injured. Otherwise, why would they claim that it's a dangerous activity?

The problem is that the Trampoline enthusiast never disagrees with that statement. In fact, they acknowledge that injuries are on the rise, and that there is risk. They just don't think the danger is so bad that we need to require professional supervision. "It's not as bad as you say" is not the same as saying there are not that many people getting hurt.

Without more info from the enthusiast, we cannot say that they would disagree with this claim, and that makes it a loser.

Get the most out of your LSAT Prep Plus subscription.

Analyze and track your performance with our Testing and Analytics Package.